SAT官方指南OG 解析7(整理版)

更新时间:2024-04-05 14:20:01 阅读量: 综合文库 文档下载

说明:文章内容仅供预览,部分内容可能不全。下载后的文档,内容与下面显示的完全一致。下载之前请确认下面内容是否您想要的,是否完整无缺。

Book Test #7:

Sample Essay - Score of 6

Everyone goes through their lives seeing and experiencing different things. At times, these different

situations may cause one to act differently than another would in the same situation. It can be readily stated that “Ease does not challenge us; adversity helps us discover who we are.” Through literature, one can see how this statement proves to be true.

In the play, “The Enemy of the People”, by Henrik Ibsen, the protagonist, Dr. Stockholm, faced adversity, and through that, was able to discover his true beliefs and who he was. Before Dr. Stockholm was put in a

situation where he had to face adversity, he had lived his life with mild ease and regularity. He acted just like everyone else in the town he lived in. Years later, he discovered that the popular bath houses was infected with bacteria. Determined to spread his findings, Dr. Stockholm went to the newspaper and the local

governing office to try and make his discovery known. However, both the paper and the government weren’t as inspired to really let the public know about this serious problem due to the popularity of the bath houses. As a result, Dr. Stockholm was forced to face adversity and find a way to achieve his previously set goal. Through adversity, Stockholm was able to truely see how his personality was. He was able to understand his severe discontent with the government and society as a whole. In essence, Stockholm was able to discover his actual self. This, however, only happened when he was faced with a challenge.

The book, “Where the Heart is” by Billie Letts also used adversity to portray it’s characters true being. The protagonist, Novalee Nation, was placed in a situation where she had to realize and conquer adversity.

Novalee was a young girl of 17, who was pregnant. She had experienced extreme hardship her whole life in the trailer park environment that she had previously resided in. She also received lots of abuse from her boyfriend who impregnated her. In the story, he had left her stranded in a K-Mart all by herself, while she was pregnant. All alone, Novalle had to find a way to keep herself and her baby alive. Along the way, through adversity, Novalle found her true, kind self. She also made countless discoveries through meeting different people. Among those things was her true love for photography and literature, and most of all her yearning for love in her life.

In the face of adversity, we all act differently. We each have different ideas and methods that we are

accustomed to use for different situations. By acting and making decisions on our own, as we are often times forced to do when given a challenge, we are able to find out who we truely are and what we truely believe in. The characters Dr. Stockholm and Novalee Nation are examples of that kind of people in literature. However, in literature and life, it is all the same. Ease does not challenge us; adversity helps us discover who we are.

Section 3

1

ANSWERS AND EXPLANATIONS Explanation for Correct Answer B :

Choice (B) is correct. It avoids the error of the original by providing a noun, \properly modified by the clause \Explanation for Incorrect Answer A :

Choice (A) involves improper modification. The opening clause (\modify a time (\Explanation for Incorrect Answer C :

Choice (C) involves an error in modification. The opening clause (\modify a time (\

Explanation for Incorrect Answer D :

Choice (D) includes a modification error. The opening clause (\modify \Explanation for Incorrect Answer E :

Choice (E) involves improper modification. The opening clause (\modify \

2

ANSWERS AND EXPLANATIONS Explanation for Correct Answer B :

Choice (B) is correct. It avoids the error of the original by providing an infinitive (\earlier infinitive \

Explanation for Incorrect Answer A :

Choice (A) includes excess words. The words \Explanation for Incorrect Answer C :

Choice (C) results in an illogical statement. It does not make sense to suggest that by agreeing \their districts,\ Explanation for Incorrect Answer D :

Choice (D) involves improper idioms. \with.\hold.\

Explanation for Incorrect Answer E :

Choice (E) distorts the intended meaning of the sentence by suggesting that it was during their trip to their districts that the legislators agreed to \3

ANSWERS AND EXPLANATIONS Explanation for Correct Answer D :

Choice (D) is correct. It avoids the error of the original by providing a main verb (\thought.

Explanation for Incorrect Answer A :

Choice (A) is a sentence fragment. There is no main verb (only the verbal \Explanation for Incorrect Answer B :

Choice (B) is a sentence fragment. There is no main verb to complete the thought.

Explanation for Incorrect Answer C :

Choice (C) results in an awkward construction that distorts the intended meaning of the sentence. The

\-part action (using kites and surveying) as the sentence suggests. Explanation for Incorrect Answer E :

Choice (E) involves is an illogical statement. The phrase \suggests that the kites alone were surveying. To properly convey the idea that the kites were used to survey enemy troops, \4

ANSWERS AND EXPLANATIONS Explanation for Correct Answer D :

Choice (D) is correct. It avoids the error of the original by providing a singular verb, \singular subject, \

Explanation for Incorrect Answer A :

Choice (A) involves an error in subject-verb agreement. The plural verb \subject \

Explanation for Incorrect Answer B :

Choice (B) involves an error in agreement. The plural verb \\

Explanation for Incorrect Answer C :

Choice (C) involves an error in subject-verb agreement. The plural verb \subject, \

Explanation for Incorrect Answer E :

Choice (E) involves an inappropriate idiom that distorts the intended meaning of the sentence. It doesn't make sense to say that people thought of a breakfast food \5

ANSWERS AND EXPLANATIONS Explanation for Correct Answer E :

Choice (E) is correct. It avoids the error of the original by removing unnecessary words. Explanation for Incorrect Answer A :

Choice (A) includes excess words. The phrase \precise \

Explanation for Incorrect Answer B :

Choice (B) makes no sense. It does not make sense to describe \Explanation for Incorrect Answer C :

Choice (C) involves an error in verb form. The verb phrase \ Explanation for Incorrect Answer D :

Choice (D) is awkward and confusing. It does not make sense to characterize \least of their colleagues.\

6

ANSWERS AND EXPLANATIONS Explanation for Correct Answer E :

Choice (E) is correct. It avoids the error of the original by properly conveying the intended comparison. Explanation for Incorrect Answer A :

Choice (A) involves an illogical comparison. \\

Explanation for Incorrect Answer B :

Choice (B) results in an illogical comparison. It is not logical to compare \and 1970's.\

Explanation for Incorrect Answer C :

Choice (C) is illogical. \ Explanation for Incorrect Answer D :

Choice (D) involves improper diction. To correctly make the comparison, the words \changed to \

7

ANSWERS AND EXPLANATIONS Explanation for Correct Answer A :

Choice (A) is correct. It avoids the errors of the other choices by properly comparing people (\architects\Explanation for Incorrect Answer B :

Choice (B) presents an illogical comparison. A thing (\(\

Explanation for Incorrect Answer C :

Choice (C) results in an illogical comparison. A thing (\compared with people (\Explanation for Incorrect Answer D :

Choice (D) presents an illogical comparison. People (\a thing (\

Explanation for Incorrect Answer E :

Choice (E) results in an illogical comparison. Things (\compared with people (\8

ANSWERS AND EXPLANATIONS Explanation for Correct Answer D :

Choice (D) is correct. It avoids the errors of the original by properly comparing \\Explanation for Incorrect Answer A :

Choice (A) involves an illogical comparison. Something that occurs \\

Explanation for Incorrect Answer B :

Choice (B) results in an illogical comparison. \happens \

Explanation for Incorrect Answer C :

Choice (C) involves an improper idiom. What comes after each part of the correlative construction \or\\

Explanation for Incorrect Answer E :

Choice (E) involves an improper idiom. What comes after each part of the correlative construction \or\preposition \

9

ANSWERS AND EXPLANATIONS Explanation for Correct Answer B :

Choice (B) is correct. It avoids the error of the original by removing excess words. Explanation for Incorrect Answer A :

Choice (A) involves wordiness. The phrase \removed.

Explanation for Incorrect Answer C :

Choice (C) involves an improper idiom. \Explanation for Incorrect Answer D :

Choice (D) is awkward and wordy. The phrase \the more precise and idiomatic \ Explanation for Incorrect Answer E :

Choice (E) is awkward and wordy. The phrase \why\10

ANSWERS AND EXPLANATIONS Explanation for Correct Answer A :

Choice (A) is correct. It avoids the errors of the other choices by clearly indicating what happened and when. Explanation for Incorrect Answer B :

Choice (B) is awkward and imprecise. Without repeating the subject by using the pronoun \does not make clear what was \Explanation for Incorrect Answer C :

Choice (C) involves improper coordination. Two complete thoughts (\signs . . . however\ Explanation for Incorrect Answer D :

Choice (D) is vague. It is not clear what the phrase \Explanation for Incorrect Answer E :

Choice (E) is awkward and imprecise. It is not clear what the phrase \refer to.

11

ANSWERS AND EXPLANATIONS Explanation for Correct Answer E :

Choice (E) is correct. It avoids the error of the original by reducing excess words (\an appropriate idiom (\

Explanation for Incorrect Answer A :

Choice (A) displays wordiness. The phrase \idiomatic \

Explanation for Incorrect Answer B :

Choice (B) exhibits unclear pronoun reference. It is not clear to what the relative pronoun \refer.

Explanation for Incorrect Answer C :

Choice (C) involves improper coordination. Two complete thoughts (\than . . . result\ Explanation for Incorrect Answer D :

Choice (D) involves improper coordination. Two complete thoughts (\number . . . reason\

12

ANSWERS AND EXPLANATIONS

Corrected Sentence: Although born in the Midwest, Langston Hughes lived most of his adult life in Harlem, in New York City, where, like other writers in the 1930's, he wrote some of his finest works. Explanation for Correct Answer C :

The error in this sentence occurs at (C), where an improper verb form is used. The past perfect tense of the verb phrase \past participle of \Explanation for Incorrect Answer A :

There is no error at (A). The subordinating conjunction \clause that modifies the proper noun \Explanation for Incorrect Answer B :

There is no error at (B). The preposition \that modifies the proper noun \Explanation for Incorrect Answer D :

There is no error at (D). The superlative adjective \of more than two things (all of Hughes' works).

Explanation for Incorrect Answer E : There is an error in the sentence. 13

ANSWERS AND EXPLANATIONS

Corrected Sentence: In constant demand as a speaker, Ms. Chernock has never been busier than she is now. Explanation for Correct Answer C :

The error in this sentence occurs at (C), where a double comparative is used. The adverb “more” is not needed to modify the already comparative adjective “busier.” Explanation for Incorrect Answer A :

There is no error at (A). The noun “demand” is appropriately used as the object of the preposition “in,” and the preposition “as” correctly introduces the adjective phrase that modifies the noun “demand.” Explanation for Incorrect Answer B :

There is no error at (B). The singular verb “has” agrees with its singular subject “Ms. Chernock,” the present perfect tense of the verb phrase appropriately indicates an action completed before the action of the dependent clause, and the adverb “never” is idiomatic. Explanation for Incorrect Answer D :

There is no error at (D). The singular verb “is” agrees with its singular subject “she,” and the adverb “now” correctly modifies “is.”

Explanation for Incorrect Answer E : There is an error in the sentence.

14

ANSWERS AND EXPLANATIONS

Corrected Sentence: During the night, there are usually two German shepherds at the warehouse to guard against robbery attempts.

Explanation for Correct Answer B :

The error in this sentence occurs at (B), where there is subject-verb disagreement. In this inverted sentence, the singular verb \Explanation for Incorrect Answer A :

There is no error at (A). The preposition \ Explanation for Incorrect Answer C :

There is no error at (C). The infinitive phrase \ial clause, and the preposition \Explanation for Incorrect Answer D :

There is no error at (D). The noun \Explanation for Incorrect Answer E : There is an error in the sentence. 15

ANSWERS AND EXPLANATIONS

Corrected Sentence: Candy manufacturers applauded the discovery by researchers that students who smell chocolate while studying and again while taking a test are able to recall more material than students not exposed to the odor of chocolate. Explanation for Correct Answer B :

The error in this sentence occurs at (B), where an improper pronoun is used to refer to people. The relative pronoun “which” is used instead of the pronoun “who” that is needed. Explanation for Incorrect Answer A :

There is no error at (A). The noun “discovery” correctly functions as the direct object of the verb “applauded,” and the preposition “by” is the idiomatic preposition to follow the word “discovery.” Explanation for Incorrect Answer C :

There is no error at (C). The adverb “again” appropriately modifies the adverbial phrase “while taking,” which, in turn, modifies the verb “smell.”

Explanation for Incorrect Answer D :

There is no error at (D). The plural verb “are” agrees with its plural subject “students,” and the adjective “able” properly functions as a subject complement modifying “students.”

Explanation for Incorrect Answer E : There is an error in the sentence. 16

ANSWERS AND EXPLANATIONS Corrected Sentence:

Explanation for Correct Answer E : There is no error in this sentence. Explanation for Incorrect Answer A :

There is no error at (A). The indefinite pronoun “One” correctly functions as the singular subject of the sentence, and the helping verb “can” agrees with its singular subject. Explanation for Incorrect Answer B :

There is no error at (B). The negative adverb “hardly” appropriately modifies the verb phrase “can … determine.”

Explanation for Incorrect Answer C :

There is no error at (C). The subordinating conjunction “which” correctly introduces the dependent noun clause that functions as the direct object of the verb phrase “can hardly determine,” and the singular verb “contributes” agrees with its subject “which.” Explanation for Incorrect Answer D :

There is no error at (D). The comparative adverb “more” is correctly used to compare two things, “skill” and “practice.”

17

ANSWERS AND EXPLANATIONS Corrected Sentence:

Explanation for Correct Answer E : There is no error in this sentence. Explanation for Incorrect Answer A :

There is no error at (A). The noun “refugees” appropriately functions as the direct object of the verb phrase “had not expected,” and the preposition “from” correctly introduces the adjective prepositional phrase that modifies “refugees.”

Explanation for Incorrect Answer B :

There is no error at (B). The infinitive “to be” is the idiomatic verbal form to follow “expected.” Explanation for Incorrect Answer C :

There is no error at (C). The conjunction “as” is appropriately used in the comparative construction “as … as,” and the plural pronoun “those” is parallel with the plural noun “refugees,” to which it is compared. Explanation for Incorrect Answer D :

There is no error at (D). The past perfect tense of the verb phrase “had seen” is consistent with the past perfect tense of the verb of the main clause. 18

ANSWERS AND EXPLANATIONS

Corrected Sentence: Although naturalists have identified six hundred different forms of the corion snail, there are actually no more than two true species within this genus. Explanation for Correct Answer B :

The error in this sentence occurs at (B), where the singular verb “is” does not agree with its plural subject, “species.”

Explanation for Incorrect Answer A :

There is no error at (A). The subordinating conjunction “Although” appropriately introduces the adverbial dependent clause.

Explanation for Incorrect Answer C :

There is no error at (C). The words “no,” “more,” and “than” combine to form an appropriate idiom. Explanation for Incorrect Answer D :

There is no error at (D). The preposition “within” correctly introduces the adverbial phrase that modifies the verb of the independent clause.

Explanation for Incorrect Answer E : There is an error in the sentence. 19

ANSWERS AND EXPLANATIONS

Corrected Sentence: The department of transportation has introduced pictorial traffic signs because drivers can react to them more quickly than to verbal ones. Explanation for Correct Answer B :

The error in this sentence occurs at (B), where a singular pronoun, “this,” is used instead of the plural pronoun “them” to refer to the plural noun “signs.” Explanation for Incorrect Answer A :

There is no error at (A). The singular verb phrase “has introduced” agrees with its singular subject, “department.

Explanation for Incorrect Answer C :

There is no error at (C). The comparative adverbial phrase “more quickly” is appropriate to compare one type of traffic sign to another.

Explanation for Incorrect Answer D :

There is no error at (D). The prepositional phrase “to verbal ones” is parallel with what should be the preceding prepositional phrase “to them,” and the plural pronoun “ones” correctly refers to the plural noun “signs” to which it refers.

Explanation for Incorrect Answer E : There is an error in the sentence. 20

ANSWERS AND EXPLANATIONS

Corrected Sentence: She claimed that the “representational” actor tries to imitate a character’s behavior, whereas the “presentational” actor attempts to reveal human behavior through self-understanding. Explanation for Correct Answer A :

The error in this sentence occurs at (A), where the present progressive tense of the verb “is having” is improperly used, resulting in a lack of parallelism with the present-tense verb in the subordinate clause, “attempts.”

Explanation for Incorrect Answer B :

There is no error at (B). The subordinating conjunction “whereas” appropriately introduces the dependent adverbial clause and indicates a contrast with the preceding dependent noun clause introduced by “that.” Explanation for Incorrect Answer C :

There is no error at (C). The infinitive verbal form “to reveal” is idiomatic and appropriately functions as the object of the verb “attempts.”

Explanation for Incorrect Answer D :

There is no error at (D). The prepositional phrase introduced by “through” correctly functions as an adverb that modifies “attempts.”

Explanation for Incorrect Answer E : There is an error in the sentence. 21

ANSWERS AND EXPLANATIONS

Corrected Sentence: Many nations, and the United Nations itself, have issued stamps that commemorate the fiftieth anniversary of the signing of the UN charter. Explanation for Correct Answer B :

The error in this sentence occurs at (B), where the singular verb phrase \compound subject \Explanation for Incorrect Answer A :

There is no error at (A). The reflexive pronoun \Nations\Explanation for Incorrect Answer C :

There is no error at (C). The relative pronoun \that modifies the noun \Explanation for Incorrect Answer D :

There is no error at (D). The prepositional phrase introduced by \

modifying the noun \Explanation for Incorrect Answer E : There is an error in the sentence. 22

ANSWERS AND EXPLANATIONS

Corrected Sentence: The tribal council’s program familiarized young people with Cherokee history, taught them tribal traditions, and gave them the opportunity to learn skills used by ancient artists. Explanation for Correct Answer C :

The error in this sentence occurs at (C), where an independent clause breaks the parallelism of the two preceding verb phrases (“familiarized…history” and “taught…traditions”). Explanation for Incorrect Answer A :

There is no error at (A). The preposition “with” after the verb “familiarized” is idiomatic. Explanation for Incorrect Answer B :

There is no error at (B). The past tense of the verb “taught” is consistent with the past tense of the other verbs in the sentence, and the plural pronoun “them” correctly refers to the plural noun “people.” Explanation for Incorrect Answer D :

There is no error at (D). The noun “opportunity” properly functions as the direct object of the verb, and the preposition “to,” after the word “opportunity,” is idiomatic.

Explanation for Incorrect Answer E : There is an error in the sentence. 23

ANSWERS AND EXPLANATIONS

Corrected Sentence: Observation of diverse animal species shows that the most successful in the struggle for survival are those which are most adaptable to changes in their world. Explanation for Correct Answer A :

The error in this sentence occurs at (A), where the plural verb “show” does not agree with the singular subject “Observation.”

Explanation for Incorrect Answer B :

There is no error at (B). The superlative adjective phrase “most successful” is appropriate for comparing all of the animal species in the world.

Explanation for Incorrect Answer C :

There is no error at (C). The indefinite pronoun “those” appropriately functions as the subject complement of the verb “are.”

Explanation for Incorrect Answer D :

There is no error at (D). The adjective “adaptable” appropriately functions as the subject complement of the verb “are,” and the preposition “to” after the word “adaptable” is idiomatic. Explanation for Incorrect Answer E : There is an error in the sentence. 24

ANSWERS AND EXPLANATIONS

Corrected Sentence: Far from having been a diehard conservative, Hoover was, some scholars now contend, the leading progressive of his day.

Explanation for Correct Answer A :

The error in this sentence occurs at (A), where the use of the adverb “away” is not idiomatic. The idiomatic phrase for this sentence is “Far from.” Explanation for Incorrect Answer B :

There is no error at (B). The proper noun “Hoover” correctly functions as the subject of the main clause, and the singular verb “was” agrees with its singular subject, “Hoover.” Explanation for Incorrect Answer C :

There is no error at (C). The adverb “now” appropriately modifies the plural verb “contend,” which agrees with its plural subject “scholars.”

Explanation for Incorrect Answer D :

There is no error at (D). The preposition “of” properly introduces the adjective phrase that modifies the noun “progressive,” and the singular pronoun “his” agrees with the singular noun “Hoover,” to which it refers. Explanation for Incorrect Answer E : There is an error in the sentence. 25

ANSWERS AND EXPLANATIONS

Corrected Sentence: For the past hundred years or more, Yellowstone National Park has been a kind of sociological laboratory in which North Americans have been exploring the meaning of the national-park concept.

Explanation for Correct Answer B :

The error in this sentence occurs at (B), where the past tense of the verb “was” is inconsistent with the present perfect progressive tense of the verb phrase “have been exploring.” Explanation for Incorrect Answer A :

There is no error at (A). The preposition “For” appropriately introduces an adverbial phrase that modifies the verb of the main clause.

Explanation for Incorrect Answer C :

There is no error at (C). The preposition “in” is an appropriate idiom to connect the noun “laboratory” to the adjective clause that describes “laboratory,” and “which” is the correct relative pronoun to introduce that clause.

Explanation for Incorrect Answer D :

There is no error at (D). The noun phrase “the meaning” appropriately functions as the direct object of the verb phrase “have been exploring,” and the preposition “of” after the word “meaning” is idiomatic. Explanation for Incorrect Answer E : There is an error in the sentence. 26

ANSWERS AND EXPLANATIONS

Corrected Sentence: Alerted by the nervousness and evasiveness of the witness, the jurors were quick to perceive that his statements were inconsistent with those he had made earlier.

Explanation for Correct Answer C :

The error in this sentence occurs at (C), where the preposition “to” after the word “inconsistent” is not idiomatic. “With” is needed instead.

Explanation for Incorrect Answer A :

There is no error at (A). The past participle “Alerted” appropriately modifies the noun “jurors,” and the preposition “by” combines with “Alerted” to form an appropriate idiom. Explanation for Incorrect Answer B :

There is no error at (B). The infinitive “to perceive” correctly functions as an adverb modifying the adjective “quick.”

Explanation for Incorrect Answer D :

There is no error at (D). The plural pronoun “those” correctly functions as the object of the preposition and appropriately refers to the plural noun “statements.”

Explanation for Incorrect Answer E : There is an error in the sentence. 27

ANSWERS AND EXPLANATIONS

Corrected Sentence: In many respects Anna Karenina and Emma Bovary are very similar characters, but Bovary has more spirit and determination. Explanation for Correct Answer D :

The error in this sentence occurs at (D), where the superlative adjective “most” is used instead of the comparative adjective “more” that is needed when comparing only two entities. Explanation for Incorrect Answer A :

There is no error at (A). The prepositional phrase “In … respects” is idiomatic, and the plural adjective “many” appropriately modifies the plural noun “respects.” Explanation for Incorrect Answer B :

There is no error at (B). The coordinating conjunction “but” connects two independent clauses and appropriately indicates a contrast between them. Explanation for Incorrect Answer C :

There is no error at (C). The singular verb “has” agrees with its singular subject “Bovary” (the second instance).

Explanation for Incorrect Answer E : There is an error in the sentence. 28

ANSWERS AND EXPLANATIONS

Corrected Sentence: Between the sales manager and me existed an easy, cooperative working relationship; neither of us hesitated to discuss problems. Explanation for Correct Answer A :

The error in this sentence occurs at (A), where an incorrect pronoun case is used. The nominative case of the first-person pronoun “I” is used where the objective case “me” is needed. Explanation for Incorrect Answer B :

There is no error at (B). The verb “existed” can be singular or plural and agrees with its singular subject, “relationship.” The past tense is consistent with the past tense of the verb in the other independent clause. Explanation for Incorrect Answer C :

There is no error at (C). The indefinite article “an” and the adjective “easy” both appropriately modify the noun “relationship.”

Explanation for Incorrect Answer D :

There is no error at (D). The indefinite singular pronoun “neither,” which refers to the two singular nouns in the preceding clause, “manager” and “me,” is an appropriate subject for the second clause. The pronoun “us” is in the objective case and correctly functions as the object of the preposition “of.” Explanation for Incorrect Answer E : There is an error in the sentence. 29

ANSWERS AND EXPLANATIONS Corrected Sentence:

Explanation for Correct Answer E : There is no error in this sentence. Explanation for Incorrect Answer A :

There is no error at (A). The preposition “As” appropriately introduces the adjective prepositional phrase that modifies the subject of the sentence, “Thomas à Becket.” Explanation for Incorrect Answer B :

There is no error at (B). The noun phrase “an independence” correctly functions as the direct object of the verb “assumed.”

Explanation for Incorrect Answer C :

There is no error at (C). The use of the preposition “to” after the word “intolerable” is idiomatic. Explanation for Incorrect Answer D :

There is no error at (D). The past perfect tense of the verb phrase “had … been” correctly indicates an action that was completed before the action of the two preceding past-tense verbs, and the adverb “long” and the verb “been” combine to form an appropriate idiom. 30

ANSWERS AND EXPLANATIONS Explanation for Correct Answer A :

Choice (A) is correct. The sentence has a clear subject and verb, \anizes the rest of the information with the word \Explanation for Incorrect Answer B :

Choice (B) is unsatisfactory because \geology\Explanation for Incorrect Answer C :

Choice (C) is unsatisfactory because it has no subject; the sentence consists of two dependent clauses. Explanation for Incorrect Answer D :

Choice (D) is unsatisfactory because it is imprecise: the resulting sentence does not clarify the relationship between the Hoovers and Stanford University. Explanation for Incorrect Answer E :

Choice (E) is unsatisfactory because it joins two independent thoughts with only a comma. 31

ANSWERS AND EXPLANATIONS Explanation for Correct Answer D :

Choice (D) is correct. The word \independent clause \ Explanation for Incorrect Answer A :

Choice (A) is unsatisfactory because it is illogical: it says that the purpose of Hoover designing the crib was for it to remain stable, not for her sons to have a place to sleep. Explanation for Incorrect Answer B :

Choice (B) is unsatisfactory because the stability of the crib was the key feature of its design; the two parts of the sentence should not be separated by \ Explanation for Incorrect Answer C :

Choice (C) is unsatisfactory because the word \does not exist in the sentence.

Explanation for Incorrect Answer E :

Choice (E) is unsatisfactory because it unnecessarily repeats the word \32

ANSWERS AND EXPLANATIONS Explanation for Correct Answer D :

Choice (D) is correct. The writer's intention is to praise Hoover, and the second paragraph is essentially a list of her accomplishments, so it makes sense in the paragraph's concluding sentence for the writer to reiterate that these details show what a \ Explanation for Incorrect Answer A :

Choice (A) is unsatisfactory because it makes an illogical link with the preceding sentence: it makes no sense to state that Hoover designed a crib \

Explanation for Incorrect Answer B :

Choice (B) is unsatisfactory because it inappropriately switches verb tenses (\published\

Explanation for Incorrect Answer C :

Choice (C) is unsatisfactory because it is unclear what time the phrase \ime\Explanation for Incorrect Answer E :

Choice (E) is unsatisfactory because \a crib and the achievements of learning languages and publishing scholarly papers that is not supported by the passage.

33

ANSWERS AND EXPLANATIONS Explanation for Correct Answer A :

Choice (A) is correct. Changing \independent clause the sentence needs, and makes verb tenses consistent throughout the sentence. Explanation for Incorrect Answer B :

Choice (B) is unsatisfactory because it fails to address the fact that the original sentence consists of two dependent clauses.

Explanation for Incorrect Answer C :

Choice (C) is unsatisfactory because the replacement of \incorrect sentence that seems to compare Lou Hoover to herself. Explanation for Incorrect Answer D :

Choice (D) is unsatisfactory because the deletion of \the sentence lacks an independent clause.

Explanation for Incorrect Answer E :

Choice (E) is unsatisfactory because, while \substitution to make in the second clause, the sentence still consists of two dependent clauses. 34

ANSWERS AND EXPLANATIONS Explanation for Correct Answer B :

Choice (B) is correct. The sentence is properly linked with the preceding sentences through the phrase \addition,\ Explanation for Incorrect Answer A :

Choice (A) is unsatisfactory because entertaining Jessie DePriest was the act that broke the racial barrier; the two parts of the sentence should not be separated by \Explanation for Incorrect Answer C :

Choice (C) is unsatisfactory because it reflects an unnecessary shift in tenses from the rest of the passage. Explanation for Incorrect Answer D :

Choice (D) is unsatisfactory because the information in sentence 14 is not a consequence of the information in sentence 13.

Explanation for Incorrect Answer E :

Choice (E) is unsatisfactory because it is illogical; this structure would only make sense if the clauses were reversed (\ 35

ANSWERS AND EXPLANATIONS Explanation for Correct Answer B :

Choice (B) is correct. Every sentence of the passage (except for the first two) praises Hoover as an

exceptionally gifted and active woman, so a concluding sentence summarizing Hoover's life in a positive way is appropriate and expected.

Explanation for Incorrect Answer A :

Choice (A) is unsatisfactory because the passage does not discuss Hoover as \of the passage is on Hoover's adventures and accomplishments. Explanation for Incorrect Answer C :

Choice (C) is unsatisfactory because the first mention of Hoover's personal papers should not come in the last sentence of a passage about her life. Explanation for Incorrect Answer D :

Choice (D) is unsatisfactory because it is not \to assume that the writer knows a great deal about Hoover's early life, but the passage focuses on her accomplishments as an adult.

Explanation for Incorrect Answer E :

Choice (E) is unsatisfactory because it makes no sense to say that historians are the only people who know the year and town of Hoover's birth. All readers of the passage now know this information.

Section 4

1

ANSWERS AND EXPLANATIONS Explanation for Correct Answer E :

Choice (E) is correct. \would read “Initially only the carpeting outside the restroom was drenched by water from the burst pipe; eventually the entire hallway flooded.” A burst pipe would spill water onto the carpet, thereby drenching it. Explanation for Incorrect Answer A :

Choice (A) is incorrect. \

sentence would read “Initially only the carpeting outside the restroom was diverted by water from the burst pipe; eventually the entire hallway flooded.” A carpet does not typically move and thus is not likely to be turned aside by water from a broken pipe. Explanation for Incorrect Answer B :

Choice (B) is incorrect. \the sentence would read “Initially only the carpeting outside the restroom was confined by water from the burst pipe; eventually the entire hallway flooded.” Water cannot generally restrict or close in something solid, such as a carpet.

Explanation for Incorrect Answer C :

Choice (C) is incorrect. \sentence would read “Initially only the carpeting outside the restroom was scuttled by water from the burst pipe; eventually the entire hallway flooded.” In order to be scuttled, an object must first be floating, usually by design. So boats can be scuttled, but carpeting, which is not designed or intended to float, cannot. Explanation for Incorrect Answer D :

Choice (D) is incorrect. \would read “Initially only the carpeting outside the restroom was cleansed by water from the burst pipe; eventually the entire hallway flooded.” Water gushing from a burst pipe would not be said to clean the carpeting. Carpets are usually ruined when they get wet from burst pipes. 2

ANSWERS AND EXPLANATIONS Explanation for Correct Answer B :

Choice (B) is correct. \attack. If one were to insert these terms into the text, the sentence would read \

advantages and disadvantages for animals like voles and mice: they enable the animals to maintain body heat, but leave them more vulnerable to discovery by predators.\describe both an advantage and a disadvantage of communal nests. Such an environment would certainly help animals to \predators, making them \Explanation for Incorrect Answer A :

Choice (A) is incorrect. \predisposed, or susceptible. If one were to insert these terms into the text, the sentence would read

\to insure body heat, but leave them more inclined to discovery by predators.\likelihood of an attack, but it makes little sense to say that body heat may be \ Explanation for Incorrect Answer C :

Choice (C) is incorrect. \insert these terms into the text, the sentence would read \

disadvantages for animals like voles and mice: they enable the animals to squander body heat, but leave them more liable to discovery by predators.\so it makes little sense to claim that communal nests \Furthermore, the sentence structure indicates that the first missing term will describe an advantage of communal nests, not a disadvantage. Explanation for Incorrect Answer D :

Choice (D) is incorrect. \one were to insert these terms into the text, the sentence would read \disadvantages for animals like voles and mice: they enable the animals to stimulate body heat, but leave them more resistant to discovery by predators.\structure of the sentence indicates that the second missing term will describe a disadvantage of communal nests, but something that makes animals more \predators is obviously advantageous. Explanation for Incorrect Answer E :

Choice (E) is incorrect. \these terms into the text, the sentence would read \animals like voles and mice: they enable the animals to retain body heat, but leave them more immune to discovery by predators.\environment that makes animals \ 3

ANSWERS AND EXPLANATIONS Explanation for Correct Answer C :

Choice (C) is correct. \sentence would read “He displayed a nearly pathological curiosity, insisting on knowing every detail of his friends’ lives.” The participial phrase \

comma signals that the phrase will explain the claim that precedes it. \person who insists on knowing every detail of his friends' lives would be displaying an unhealthy curiosity. Explanation for Incorrect Answer A :

Choice (A) is incorrect. \into the text, the sentence would read “He displayed a nearly pathological orderliness, insisting on knowing every detail of his friends’ lives.” The phrase following the comma is positioned in the sentence as if it were an explanation of what is meant by \insisting on knowing every detail of the lives of one's friends is not orderliness. Explanation for Incorrect Answer B :

Choice (B) is incorrect. \the text, the sentence would read “He displayed a nearly pathological credulity, insisting on knowing every detail of his friends’ lives.” The word \ay; it has nothing to do with asking for information. However, the position of the phrase \every detail of his friends’ lives\of the statement preceding the comma. Explanation for Incorrect Answer D :

Choice (D) is incorrect. \text, the sentence would read “He displayed a nearly pathological shyness, insisting on knowing every detail of his friends’ lives.” The participial phrase beginning with \sentence in a way that implies that it should provide a helpful explanation for the statement about shyness, which it does not.

Explanation for Incorrect Answer E :

Choice (E) is incorrect. \were to insert this term into the text, the sentence would read “He displayed a nearly pathological morbidity, insisting on knowing every detail of his friends’ lives.” Despite the implication made by the placement in the sentence of the participial phrase beginning with \help to explain what is meant by \

4

ANSWERS AND EXPLANATIONS Explanation for Correct Answer E :

Choice (E) is correct. \were to insert these terms into the text, the sentence would read \this mosquito-borne disease continues to flourish: the World Health Organization estimates that it still affects up to 500 million people a year.\between \contrast the global efforts to \Furthermore, the idea that malaria \supported by the phrase following the colon, which indicates that a large number of people are still affected by the disease. Explanation for Incorrect Answer A :

Choice (A) is incorrect. \insert these terms into the text, the sentence would read \

mosquito-borne disease continues to flag: the World Health Organization estimates that it still affects up to 500 million people a year.\sentence sets up a contrast between \efforts\ flag\people\Explanation for Incorrect Answer B :

Choice (B) is incorrect. \were to insert these terms into the text, the sentence would read \to foster malaria, this mosquito-borne disease continues to thrive: the World Health Organization estimates that it still affects up to 500 million people a year.\fostering them. In addition, the expectation set up by the word \clause is consistent with, and does not contrast sharply with, that of the second clause. The normal expectation is that things that are fostered will thrive. Explanation for Incorrect Answer C :

Choice (C) is incorrect. \these terms into the text, the sentence would read \

mosquito-borne disease continues to abate: the World Health Organization estimates that it still affects up to 500 million people a year.\that follows it. However, efforts to combat malaria would normally result in an \the disease, so the expectation set up by \Explanation for Incorrect Answer D :

Choice (D) is incorrect. \insert these terms into the text, the sentence would read \mosquito-borne disease continues to prosper: the World Health Organization estimates that it still affects up to 500 million people a year.\ of a disease. Therefore, it is not surprising that the efforts to scrutinize malaria did not prevent the disease from prospering. However, the word \contrast will be described.

5

ANSWERS AND EXPLANATIONS Explanation for Correct Answer B :

Choice (B) is correct. \would read \innocuous and unlikely to offend.\The word \the second clause will provide a statement that contrasts strongly with the assertion made in the first clause. Because the review panel's condemnation of the movie is contrasted with critic Kael's belief that the movie is harmless, \ Explanation for Incorrect Answer A :

Choice (A) is incorrect. \insert this term into the text, the sentence would read \Pauline Kael the movie seemed entirely impressionable and unlikely to offend.\applies to people, not things. So the movie, although it can make an impression on people, cannot be said to be impressionable.

Explanation for Incorrect Answer C :

Choice (C) is incorrect. \ert this term into the text, the sentence would read \seemed entirely unsuitable and unlikely to offend.\meaning will contrast with the main clause of the sentence; that does not happen here because the

condemnation by the review panel is compatible with, not in opposition to, Kael's impression that the movie is unsuitable.

Explanation for Incorrect Answer D :

Choice (D) is incorrect. \to insert this term into the text, the sentence would read \critic Pauline Kael the movie seemed entirely insensitive and unlikely to offend.\very likely to offend, so the sentence does not make sense with that term. Explanation for Incorrect Answer E :

Choice (E) is incorrect. \term into the text, the sentence would read \seemed entirely unapproachable and unlikely to offend.\the expectation in the reader's mind that its meaning will contrast with the main clause of the sentence. Being unapproachable, however, does not contrast with the sorts of qualities that would cause a review board to condemn a movie.

6

ANSWERS AND EXPLANATIONS Explanation for Correct Answer B :

Choice (B) is correct. \term into the text, the sentence would read \Abdalla’s autobiography as a portrait of an indomitable person, one who prevailed against great odds.\succeeds, against great odds. Because of the structure of the sentence, the reader expects such a definition or explanation to follow the comma.

Explanation for Incorrect Answer A :

Choice (A) is incorrect. \s term into the text, the sentence would read \alla’s

autobiography as a portrait of an empathetic person, one who prevailed against great odds.\clearly refers back to \

comma will provide an appropriate definition or explanation of \beginning with \Explanation for Incorrect Answer C :

Choice (C) is incorrect. \the sentence would read \Abdalla’s autobiography as a portrait of an expeditious person, one who prevailed against great odds.\well and fast under normal circumstances but might not necessarily be up to the task of \against great odds. The position of the phrase after the comma in the sentence leads experienced readers to expect that the phrase will provide a helpful definition or explanation of \not happen here.

Explanation for Incorrect Answer D :

Choice (D) is incorrect. \the sentence would read \Abdalla’s autobiography as a portrait of an idiosyncratic person, one who prevailed against great odds.\one who prevails against great odds. The structure of the sentence leads the reader to expect that the phrase following the comma will either define or help explain what is meant by \not happen here.

Explanation for Incorrect Answer E :

Choice (E) is incorrect. \sentence would read \Abdalla’s autobiography as a portrait of an astute person, one who prevailed against great odds.\capable of prevailing against great odds. This does not meet the expectation set up by the structure of the sentence that the phrase \ 7

ANSWERS AND EXPLANATIONS Explanation for Correct Answer A :

Choice (A) is correct. \these terms into the text, the sentence would read \company was actually insolvent and met its financial obligations only by engaging in fraudulent activities.\word \clause. This is the case here because it is somewhat surprising that a company that is bankrupt could remain in business for several months. The second clause provides one way, a dishonest one, that explains how a bankrupt company might manage to meet its financial obligations. Explanation for Incorrect Answer B :

Choice (B) is incorrect. \insert these terms into the text, the sentence would read \the company was actually prudent and met its financial obligations only by engaging in speculative activities.\A company that is \ Explanation for Incorrect Answer C :

Choice (C) is incorrect. \one were to insert these terms into the text, the sentence would read \stayed in business for several months, the company was actually autonomous and met its financial obligations only by engaging in

subordinate activities.\and one that can stay in business for several months. The word \that the meaning of the first clause will clearly contrast with that of the second clause.

Explanation for Incorrect Answer D :

Choice (D) is incorrect. \If one were to insert these terms into the text, the sentence would read \several months, the company was actually bankrupt and met its financial obligations only by engaging in

charitable activities.\say that giving to charities would help a bankrupt company pay its debts.

Explanation for Incorrect Answer E :

Choice (E) is incorrect. \If one were to insert these terms into the text, the sentence would read \several months, the company was actually stable and met its financial obligations only by engaging in

manipulative activities.\surprising or unexpected with respect to the meaning of the main clause of the sentence. However, there is nothing surprising about a stable company that remains in business. 8

ANSWERS AND EXPLANATIONS Explanation for Correct Answer B :

Choice (B) is correct. \text, the sentence would read \Frankenstein centers on a scientist’s hubris, the overweening pride that makes him believe he can usurp nature.\a definition or explanation of the phrase that precedes the comma. \that would lead a scientist to believe that he can \elonging to nature. Explanation for Incorrect Answer A :

Choice (A) is incorrect. \sentence would read \Frankenstein centers on a scientist’s obstinacy, the overweening pride that makes him believe he can usurp nature.\of the sentence suggests that the missing word will be synonymous with the phrase \try to accomplish a seemingly impossible task, but stubbornness is not the same thing as pride. Explanation for Incorrect Answer C :

Choice (C) is incorrect. \thinking. If one were to insert this term into the text, the sentence would read \Frankenstein centers on a scientist’s impetuosity, the overweening pride that makes him believe he can usurp nature.\of the sentence, however, leads the reader to expect that a definition or explanation of \the comma.

Explanation for Incorrect Answer D :

Choice (D) is incorrect. \this term into the text, the sentence would read \Frankenstein centers on a scientist’s valor, the overweening pride that makes him believe he can usurp nature.\synonymous with the phrase \would need courage, but courage and pride are not synonyms. Explanation for Incorrect Answer E :

Choice (E) is incorrect. \term into the text, the sentence would read \Frankenstein centers on a scientist’s callousness, the

overweening pride that makes him believe he can usurp nature.\Readers would expect an explanation of \

9

ANSWERS AND EXPLANATIONS Explanation for Correct Answer B :

Choice (B) is correct. The author of Passage 2 speaks specifically of Clemens and claims that,\of Samuel Clemens, the death of his daughter, and the chronic illness of his wife are agonizing as personal history\\(lines 5-6). Both authors agree that Clemens endured painful personal loss. Explanation for Incorrect Answer A :

Choice (A) is incorrect. The author of Passage 1 is clear about Clemens' deep despair over \discords, moral conflicts, and economic problems\his personal life. The author of Passage 2, however, does not comment on Clemens' views of society and humanity. Passage 2 focuses on the question of whether, and how, Clemens' personal disasters were transformed into written works of art. Explanation for Incorrect Answer C :

Choice (C) is incorrect. Although Passage 2 refers to literary critics (lines 17-19), neither passage contains any information about whether Clemens was affected by literary critics. Explanation for Incorrect Answer D :

Choice (D) is incorrect. Passage 1 discusses Clemens and Adams together when it refers to \

tragedies\endured by one were much like those endured by the other. However, the author of Passage 2 does not

mention Adams. So there is no indication that the author of Passage 2 believes that Adams endured hardships like those Clemens endured.

Explanation for Incorrect Answer E :

Choice (E) is incorrect. The author of Passage 1 says that \despair is in a slow process of incubation from their earliest work\1 seems to believe that Clemens \of Passage 2 does not comment on indications of pessimism in Clemens' work. Thus it is not appropriate to claim that \agree\ 10

ANSWERS AND EXPLANATIONS Explanation for Correct Answer B :

Choice (B) is correct. By using the metaphor of \about Adams' and Clemens' growing despair, the author of Passage 1 suggests gradual development. According to the argument in Passage 1, slowly growing despair is a factor over the entire span of both Clemens' and Adams' creative careers. Explanation for Incorrect Answer A :

Choice (A) is incorrect. The biological processes of \steady progression that does not seem to be strongly associated with literary creativity. In addition, although \despair evolved.

Explanation for Incorrect Answer C :

Choice (C) is incorrect. The phrase \

exceptional. There is nothing rare or exceptional about an egg, its incubation and, finally, the hatching of a baby bird.

Explanation for Incorrect Answer D :

Choice (D) is incorrect. \that something was originally unacknowledged, but gradually came to be accepted. The passage does state that \of incubation\(lines 7-8); however, there is no sense that the writers' despair was originally unacknowledged. Explanation for Incorrect Answer E :

Choice (E) is incorrect. The metaphor of incubation and hatching actually could be taken to suggest a

\the metaphor is meant to shed light on is the writers' growing despair. There is nothing about this situation that resembles a culminating achievement. 11

ANSWERS AND EXPLANATIONS Explanation for Correct Answer E :

Choice (E) is correct. What the author of Passage 1 refers to as \tragedies that Clemens (as well as Adams) suffered. The author of Passage 2 mentions three such tragedies in Clemens' life: \wife\\ Explanation for Incorrect Answer A :

Choice (A) is incorrect. The \personal experiences that, according to the author of Passage 2, significantly influenced Clemens' literary output. The author of Passage 2 calls Clemens' book The Mysterious Stranger \and claims that it represents Clemens' successful effort to transform his agonizing personal experiences into art.

Explanation for Incorrect Answer B :

Choice (B) is incorrect. The personal experiences that the author of Passage 1 calls \6) of Clemens' despair are the same experiences that Clemens, according to Passage 2, struggles to transform into art from 1895 onward. Clemens was born in 1835, so these experiences affect him as a creative artist late rather than early in his career.

Explanation for Incorrect Answer C :

Choice (C) is incorrect. The personal experiences that the author of Passage 1 calls \6) were obviously very important to Clemens. In fact, the author of Passage 2 calls these experiences \important\personal experiences with the importance of political, moral, and economic factors. Explanation for Incorrect Answer D :

Choice (D) is incorrect. The author of Passage 1 describes Clemens' personal disasters as \(line 6). The author of Passage 2 presents these \critics (particularly in the way that they contribute to the development of a work of art). Critics are particularly

interested in exploring Clemens' personal tragedies because it is one of the rare cases in which they can watch his attempts and final success in transforming his experiences into a work of art. 12

ANSWERS AND EXPLANATIONS Explanation for Correct Answer D :

Choice (D) is correct. The author of Passage 1 is interested in accounting for the growing mood of despair in the works of Adams and Clemens. The author's conclusion is that, \of country—and ultimately of humanity—that manifests itself in their works\author, this despair of country was brought on by \problems\the case of both Adams and Clemens, the author of Passage 1 suggests that it was not personal history, but public events, that asserted the strongest influence on their writings. Explanation for Incorrect Answer A :

Choice (A) is incorrect. The author of Passage 1 points to a connection between the lives of Adams and Clemens; both \Passage 1 does not suggest that this knowledge is essential to readers of either writer. Explanation for Incorrect Answer B :

Choice (B) is incorrect. According to the author of Passage 1, Clemens and Adams traveled similar paths to total despair, for similar reasons. The passage does not compare and contrast the difficulties the two writers faced. In particular, it does not address the issue of which of the two had the more distressing personal difficulties.

Explanation for Incorrect Answer C :

Choice (C) is incorrect. The author of Passage 1 mentions \

presumably the same misfortunes the author of Passage 2 describes as \Further, the author of Passage 1 suggests that these \explain the two writers' states of despair. Although the author of Passage 1 suggests that it is despair of

country, rather than \indication that the author of Passage 1 would regard the \\with the tone and character\Explanation for Incorrect Answer E :

Choice (E) is incorrect. The author of Passage 1 does not describe Clemens as a \fact, in the aspects of their lives that Passage 1 focuses on, Clemens is clearly not unique, because the development of his despair and the explanation for it are no different from what they are for Adams. 13

ANSWERS AND EXPLANATIONS Explanation for Correct Answer E :

Choice (E) is correct. The passage begins with an account of the author's personal struggle to adjust to the temporary immobilization of his right arm following surgery. The passage then opens out to broad reflections on the nature of the brain and its ability to adapt, which leads the author to promote a \dynamic and active\ Explanation for Incorrect Answer A :

Choice (A) is incorrect. The author does not present any scientific evidence that might refute a particular view. The author claims to have observed \view, but he does not say what those radical adaptations are and how exactly they support his view. Explanation for Incorrect Answer B :

Choice (B) is incorrect. The passage does not contain any amusing anecdotes. One reference that can be

described as anecdotal is the author saying that \19-20), but this is not meant to be amusing. Neither are the anecdotes \they illustrate the author's main point that the brain is remarkably able to adapt to changing conditions. Explanation for Incorrect Answer C :

Choice (C) is incorrect. The paragraphs that follow the author's observations regarding his personal story of recovery are not in the least detached. In fact, the general viewpoint that the author develops here is repeatedly grounded in the author's own experience. For example, the author says, \

development or disease is something I see, potentially, in almost every patient\of the brain's remarkable plasticity . . . has come to dominate my own perception of my patients and their lives\(lines 68-72).

Explanation for Incorrect Answer D :

Choice (D) is incorrect. A \a precisely formulated, testable claim expressing an inference from observed data. The passage does not offer an intensive analysis of the author's recovery, only a loose collection of suggestive observations. It also does not state a scientific hypothesis. 14

ANSWERS AND EXPLANATIONS Explanation for Correct Answer C :

Choice (C) is correct. The author describes himself as \-handed\situation in which he is unable to use his right arm. Emphasizing his right-handedness in this way serves to show the tremendous challenge the author faced when his right arm was immobilized. Obviously, the more strongly established the habit of using one arm in preference to the other, the more difficult it is to adapt to being without that arm.

Explanation for Incorrect Answer A :

Choice (A) is incorrect. The author does not mention being \point about how easy it was for him to become ambidextrous. For one thing, he has not become ambidextrous. For example, even though he is able to write using his left hand, he writes \

Moreover, to the extent that he is learning to use his left hand and arm, the process is not an easy one. As part of the evidence of how difficult this process has been, the author writes that \finger on my left hand\Explanation for Incorrect Answer B :

Choice (B) is incorrect. The author does not use his being \-handed\

comparing his particular abilities with those of other individuals. The contrast the author is primarily suggesting is between how he did things before (although this is not actually described) and after the arm surgery: \awkwardly\ Explanation for Incorrect Answer D :

Choice (D) is incorrect. Readers will probably be sympathetic to the difficulties faced by the author. But the author does not mention his strong right-handedness to gain sympathy. Throughout the passage, the author

is matter-of-fact (\seem to want readers to feel sorry for him. The point of mentioning his strong right-handedness is to give a sense of the magnitude of the adjustment that the author had to make. Explanation for Incorrect Answer E :

Choice (E) is incorrect. The phrase \the right hand is, or was, particularly strong. The phrase means that there is a strong tendency to use the right hand instead of the left hand.

15

ANSWERS AND EXPLANATIONS Explanation for Correct Answer A :

Choice (A) is correct. \going on with some of the programs and circuits in my brain\imaging are still too crude to show these)\of his own observations. Because his theory is impossible to confirm, it is best described as conjecture. Explanation for Incorrect Answer B :

Choice (B) is incorrect. The author's remark, which includes a reference to \s in my brain\(lines 14-15), uses the language of computers (programs, circuits) metaphorically to talk about the brain, but this is not irony. \author's remark in lines 14-16 contains no irony of any kind. Explanation for Incorrect Answer C :

Choice (C) is incorrect. \as suggesting an inquiry or investigation, though the author seems to indicate that the proper technology with which to conduct an inquiry is not currently available (lines 16-17). However, this does not mean that the remark itself is an inquiry. The remark is a statement of what the author has come to believe. Explanation for Incorrect Answer D :

Choice (D) is incorrect. Although the author's remark in lines 14-16 is based on his observations, the remark itself is not an observation. An observation is based on physical data. This remark goes one step beyond simply offering physical data by creating a theory to explain what has been observed. Explanation for Incorrect Answer E :

Choice (E) is incorrect. The author's remark is not, in itself, \here consists entirely of the author's observations of himself. Further, the author explains that \of brain imaging are still too crude\-17). The more basic point is that the author's remark in lines 14-16 does not present evidence; rather, it presents an inference based on that evidence.

16

ANSWERS AND EXPLANATIONS Explanation for Correct Answer D :

Choice (D) is correct. In lines 15-16 the author mentions alterations in \signals.\methods are still \developed, to show the alterations. So in this context, \

Explanation for Incorrect Answer A :

Choice (A) is incorrect. It would be difficult to make sense of a claim that a method was too \straightforward, to show certain difficult-to-detect phenomena. Explanation for Incorrect Answer B :

Choice (B) is incorrect. There are uses of the word \a case in point. The sense of \brain imaging are not things that occur in nature, so describing methods of brain imaging as \sense of natural is inappropriate.

Explanation for Incorrect Answer C :

Choice (C) is incorrect. A \be considered offensive. In lines 16-17, the point made about methods of brain imaging, however, is a point about what can be accomplished using those methods, not a point about their social acceptability. So \as used in line 17 does not mean offensive. Explanation for Incorrect Answer E :

Choice (E) is incorrect. \misuse of the term.

17

ANSWERS AND EXPLANATIONS Explanation for Correct Answer E :

Choice (E) is correct. The author opens the passage with a description of how he adapted to having his right arm immobilized following surgery. The author explains that most of the adaptations \

themselves, unconsciously, by reprogrammings and adaptations of which I know nothing\ 20-22). The author indicates that these physical adaptations are as far outside his conscious knowledge as \walk.\ Explanation for Incorrect Answer A :

Choice (A) is incorrect. By saying that he does not know how he normally walks, the author is not depicting his physical capabilities before his accident. What he is saying is that normal walking is not an activity that is controlled by the conscious mind. The purpose of saying this is to tell the reader that the author's newly adapted ways of doing things are no different from normal ways of doing things. Explanation for Incorrect Answer B :

Choice (B) is incorrect. The author's way of referring to learning from experience is learning \error%unconscious adaptations. He does not seem to be very interested in the first kind, and the author’s

parenthetical reference in lines 22-23 serves to highlight a point about unconscious adaptations, not about learning through experience.

Explanation for Incorrect Answer C :

Choice (C) is incorrect. The author says, \—not merely this left-handed writing, but a dozen other left-handed skills as well\author continues to be unable to perform simple tasks. It suggests that at this point the author might well be able to perform simple tasks again, only with the left hand rather than with the right. Since no inability to perform simple tasks has been established, there is none to explain.

Explanation for Incorrect Answer D :

Choice (D) is incorrect. To say that the author rationalizes a frustration presupposes that there is a frustration to rationalize. It is possible that the author was frustrated early on in his recovery, although he does not say so. At this point, the author just seems intensely interested in what is happening to him and observes it closely. He does not seem to be disappointed or discouraged. 18

ANSWERS AND EXPLANATIONS Explanation for Correct Answer D :

Choice (D) is correct. The surgeon's remarks in lines 34-36 are concerned with the author's recovery from arm surgery. With respect to the path to recovery to be taken, the surgeon tells the author: \general guidelines, restrictions, recommendations. But all the particulars you will have to find out for yourself.\words, there is a broad framework within which recovery will take place, but for the most part the patient will have to develop his own procedures for coping. Explanation for Incorrect Answer A :

Choice (A) is incorrect. The surgeon does not mention muscular adjustments at all. The author does mention them (line 30) but only to say that they will be part of his recovery from surgery. There is no indication that the author is insufficiently open-minded about muscular adjustments, so the surgeon would not appear to have any reason to advise him to become more open-minded about them. Explanation for Incorrect Answer B :

Choice (B) is incorrect. What the surgeon tells the author, according to lines 34-36, is basically that there will be no detailed and specific regimen of rehabilitation for the author to follow. There will be general rules, but beyond that, the author will have to find his own way. Explanation for Incorrect Answer C :

Choice (C) is incorrect. The surgeon tells the author that there are general guidelines that are common to everyone who has undergone the kind of surgery the author has. So there would be no point in asking others about their adherence to those guidelines. As for the individual details of the author's recovery, the surgeon says that the author will have to figure things out for himself. The suggestion is that the details will be different for each individual.

Explanation for Incorrect Answer E :

Choice (E) is incorrect. The surgeon does not advise the author simply to wait patiently until he has recovered. The surgeon will give him some general directions, but aside from that, the surgeon's advice to the author is that he find out what works best for him personally. 19

ANSWERS AND EXPLANATIONS Explanation for Correct Answer C :

Choice (C) is correct. In his remarks the physiotherapist first makes the point that the adaptations of the nervous system are unique to each person and then adds, addressing the author, \—you must see this all the time\with the nervous system's diversity of adaptations. The physiotherapist could not reason this way unless he assumed that all neurologists (including this particular neurologist) are familiar with the nervous system's adaptability.

Explanation for Incorrect Answer A :

Choice (A) is incorrect. The physiotherapist says that the nervous system's adaptations are peculiar to each individual. Since a patient's recovery involves adaptations by the nervous system, this means that patients will differ from one another in the way they recover. However, just because the recovery processes differ from one patient to another does not mean that patients have any control over them. Thus the physiotherapist's remarks do not show that he assumes that patients have complete control over their recovery. Explanation for Incorrect Answer B :

Choice (B) is incorrect. The physiotherapist says that each person's adaptation by the nervous system \a different path\seems to assume that each neurologist understands this. But no assumption is being made about how neurologists come to have this understanding. Explanation for Incorrect Answer D :

Choice (D) is incorrect. The physiotherapist's remarks (lines 36-39) revolve around a single point: \follows a different path in each person. The nervous system creates its own paths\

physiotherapist takes it for granted that his patient, a neurologist, is familiar with this point; he says to his patient, \makes any particular assumptions about his patient's knowledge of the intricacies of the muscular system. Explanation for Incorrect Answer E :

Choice (E) is incorrect. There is nothing in the physiotherapist's remarks to suggest that he assumes that neurologists consider both healthy and injured brain processes to be parallel. Rather than parallels among adaptations, he stresses the uniqueness of each nervous system's adaptation. He does not invoke the distinction between healthy and injured brains in any way. 20

ANSWERS AND EXPLANATIONS Explanation for Correct Answer D :

Choice (D) is correct. In line 42, physicist Freeman Dyson is reported to have spoken with awe of \in the physical and biological worlds.\that is, as a brief explanation of a possibly obscure expression. This phrase is \forms and forms of life\variety.

Explanation for Incorrect Answer A :

Choice (A) is incorrect. The word \\he physical world. But more importantly, the passage itself fixes the intended meaning of the word as diversity (of forms). The phrase that spells out the meaning of \forms of life.\

Explanation for Incorrect Answer B :

Choice (B) is incorrect. The reference to \occurs in the phrase \than ours\ Explanation for Incorrect Answer C :

Choice (C) is incorrect. What is being described is the richness of \n\in the physical and biological worlds\

(lines 42-43). So the focus here is on abundance in what there is to be observed in nature, not abundance in how many different things it means. \not to emphasize an abundance of interpretations. Explanation for Incorrect Answer E :

Choice (E) is incorrect. According to the passage, \ to \and forms of life\things, but \ 21

ANSWERS AND EXPLANATIONS Explanation for Correct Answer A :

Choice (A) is correct. In the paragraph that includes lines 43-48, the author includes quotations from his surgeon and his physiotherapist, and gives a thumbnail sketch of physicist Freeman Dyson's views on

\study of nature's richness, which he says is shaped by the fact that he is a physician. The reference to his professional qualifications thus explains why he is addressing the subject of health and disease, and emphasizes the validity of his point of view. Explanation for Incorrect Answer B :

Choice (B) is incorrect. In lines 43-48 the author explains how he, as a physician, thinks that the rich diversity of nature ought to be studied. This is a statement of a broad vision for study in the future. The author does not cite any information that might serve as an illustration of the limits of scientific knowledge. Explanation for Incorrect Answer C :

Choice (C) is incorrect. The view that the author outlines in lines 43-48 does represent a physician's

perspective, but it is quite broad. It is not confined to what would be of interest to, or be of potential benefit to, his own patients. The author's view is very general. It is is about \general, and it is concerned with how \ Explanation for Incorrect Answer D :

Choice (D) is incorrect. Lines 43-48 do not establish that the author himself performs surgery. The author is presented as a \all neurologists are neurosurgeons, and there is no reference anywhere in the passage to the author as a surgeon.

Explanation for Incorrect Answer E :

Choice (E) is incorrect. The author's credentials as a neurologist are not in doubt, nor is there any indication that the author thinks they might be in doubt. Moreover, the author would probably realize that mentioning the fact that he is a physician would not go very far towards establishing specific credentials as a neurologist. Finally, the broad orientation that the author outlines in lines 43-48 goes well beyond the concerns of neurology.

22

ANSWERS AND EXPLANATIONS Explanation for Correct Answer B :

Choice (B) is correct. In lines 49-54 the author points out that there can be a creative aspect to disease. He concedes that disease can \

process it \before, and so \change can be stimulated by disease as the nervous system adapts to the consequences of disease. Explanation for Incorrect Answer A :

Choice (A) is incorrect. The only reference to anything psychological in lines 49-54 is the word \49). The primary purpose of the discussion in lines 49-54 is to point out that, from the point of view of neurology, there can be positive responses to disease. Explanation for Incorrect Answer C :

Choice (C) is incorrect. Readers may already view disease as an inevitable fact of human existence, so there may not be much point in encouraging them to take this view. Moreover, lines 49-54 certainly do not give any such encouragement. What lines 49-54 do encourage the reader to do is consider seriously the possibility that disease may play a creative role with respect to the nervous system, and not just a destructive one. Explanation for Incorrect Answer D :

Choice (D) is incorrect. The passage does not concern itself with evolution in the sense of genetic shifts in a population or a species through successive generations. It does use the term \an individual nervous system, though, and here the word carries a sense of positive change, growth, or

development. Evolution in this sense is presented as possibly being forced on the nervous system by disease (lines 50-54), rather than as being blocked, or retarded, by disease. Explanation for Incorrect Answer E :

Choice (E) is incorrect. Lines 49-54 do not address the question of how disease ought to be dealt with. The author has a clear view of how disease is, in fact, dealt with by the nervous system. The nervous system suffers losses: \system also makes gains, by having \author has made clear previously (lines 20-23), is mostly a matter of neural adaptations that occur on an unconscious level. The author does not bring spiritual matters into the discussion at all. 23

ANSWERS AND EXPLANATIONS Explanation for Correct Answer C :

Choice (C) is correct. The author's main point about \of the brain as dynamic and active rather than programmed and static\that there are others who view the brain as programmed and static, that is, as inflexible and unchanging. Explanation for Incorrect Answer A :

Choice (A) is incorrect. The author discusses adaptations as a response to at least three different sorts of situation: surgery (as in his own case), disease, and developmental disorders. He also speaks of adaptations to \response to a wider range of circumstances than just disease. There is no indication anywhere in the passage that there are others who disagree with him on this point. Explanation for Incorrect Answer B :

Choice (B) is incorrect. The passage leaves no doubt that all competent observers agree that neurological change can occur as a result of disease and developmental disorders. Disease and developmental disorders are not ordinarily considered evolutionary processes, nor does the author suggest that he or anyone else thinks they are. The author speaks of certain neurological changes as resulting in \

\not that neurological changes are an effect of evolutionary processes. Explanation for Incorrect Answer D :

Choice (D) is incorrect. The author says in lines 62-64, \are hundreds of tiny areas crucial for every aspect of perception and behavior.\the general knowledge base that neurologists share. Explanation for Incorrect Answer E :

Choice (E) is incorrect. The author says that among the many aspects of perception and behavior that depend on tiny areas in the brain is \-66). So the idea that the intellectual orientation of the individual is an organic function is presented as a mere possibility, not as a fact that people either recognize or fail to recognize. Moreover, this idea is not particularly related to the author's discussion of radical adaptations in lines 56-62, but rather to the author's presentation of a fact about brain structure (lines 62-66). 24

ANSWERS AND EXPLANATIONS Explanation for Correct Answer E :

Choice (E) is correct. The author articulates the main point of the passage in lines 54-62. The central point is that the brain is \to the needs of the organism\this point when he says that a \brain's remarkable plasticity, its capacity for the most striking adaptations\dominate the way he sees his patients and their lives. Explanation for Incorrect Answer A :

Choice (A) is incorrect. The author does mention certain relatively technical aspects of how the brain functions. For example, he notes that in the brain \and behavior\position on the extent of the brain's ability to adapt, not to \function.\

Explanation for Incorrect Answer B :

Choice (B) is incorrect. The author uses the surgery done on his right arm as a starting point for his reflections on the nervous system and the brain. This surgery is used as an example; it is not the main focus of the

passage. In fact, by the time the author reaches his basic statement of his main point (lines 56-62), surgery is no longer specifically mentioned.

Explanation for Incorrect Answer C :

Choice (C) is incorrect. The author mentions healing in what is essentially an aside, referring to \process like tissue healing\a role for physical rehabilitation in recovery. However, most of the discussion in the passage is concerned with \the result of deliberate intervention (such as physical rehabilitation) or natural healing. His main concern is with what, if any, limits there are on adaptations. Explanation for Incorrect Answer D :

Choice (D) is incorrect. Early in the passage the author speculates about what is going on in his brain as he recovers from surgery on his right arm. He reasons that, \programs and circuits in my brain—altering synaptic weights and connectivities and signals\

this he adds, in parentheses, \16-17). However, this does not suggest that the main point of the passage is concerned with the technology used to measure brain activity, particularly since there are no other mentions of this technology in the remainder of the passage.

Section 6

1

ANSWERS AND EXPLANATIONS Explanation for Correct Answer D :

Choice (D) is correct. \insert these terms into the text, the sentence would read \when Poetry magazine published her early sonnets, to 1994, when her last collection of poems came out.\information following the colon indicates that both missing terms will address the length of Sarton's career. The fact that her presence in the publishing scene \Explanation for Incorrect Answer A :

Choice (A) is incorrect. \were to insert these terms into the text, the sentence would read \

developed from 1929, when Poetry magazine published her early sonnets, to 1994, when her last collection of poems came out.\ Explanation for Incorrect Answer B :

Choice (B) is incorrect. \to insert these terms into the text, the sentence would read \from 1929, when Poetry magazine published her early sonnets, to 1994, when her last collection of poems came out.\\claim that her body of work was \Explanation for Incorrect Answer C :

Choice (C) is incorrect. \were to insert these terms into the text, the sentence would read \from 1929, when Poetry magazine published her early sonnets, to 1994, when her last collection of poems came out.\suggests that it steadily \ Explanation for Incorrect Answer E :

Choice (E) is incorrect. \these terms into the text, the sentence would read \sedate career: it soared from 1929, when Poetry magazine published her early sonnets, to 1994, when her last collection of poems came out.\that \ 2

ANSWERS AND EXPLANATIONS Explanation for Correct Answer C :

Choice (C) is correct. \insert these terms into the text, the sentence would read \

phenomenal dive of over 4,700 feet confirmed earlier speculations that such whales were among the sea’s deepest divers.\claim. Data proving the bottlenose whale's diving ability would indeed \that these animals are \ Explanation for Incorrect Answer A :

Choice (A) is incorrect. \If one were to insert these terms into the text, the sentence would read \whale’s phenomenal dive of over 4,700 feet refuted earlier theories that such whales were among the sea’s deepest divers.\conjectures that the animals are superior divers. Explanation for Incorrect Answer B :

Choice (B) is incorrect. \ons about future outcomes. If one were to insert these terms into the text, the sentence would read \recording a bottlenose whale’s phenomenal dive of over 4,700 feet challenged earlier predictions that such whales were among the sea’s deepest divers.\bottlenose whale is a remarkable diver. Explanation for Incorrect Answer D :

Choice (D) is incorrect. \were to insert these terms into the text, the sentence would read \ent data recording a bottlenose whale’s phenomenal dive of over 4,700 feet validated earlier disclaimers that such whales were among the sea’s

deepest divers.\notion that these animals are \ Explanation for Incorrect Answer E :

Choice (E) is incorrect. \

uncertainties. If one were to insert these terms into the text, the sentence would read \a bottlenose whale’s phenomenal dive of over 4,700 feet substantiated earlier doubts that such whales were among the sea’s deepest divers.\ntiate,\that bottlenose whales are among the sea's deepest divers. 3

ANSWERS AND EXPLANATIONS Explanation for Correct Answer D :

Choice (D) is correct. \hasty. If one were to insert these terms into the text, the sentence would read \circumscribed lives; no one indulged in wild or impulsive behavior.\missing term will contrast with \

compatible with \or restricted, lives would not be expected to engage in \Explanation for Incorrect Answer A :

Choice (A) is incorrect. \If one were to insert these terms into the text, the sentence would read \rambunctious lives; no one indulged in wild or indecent behavior.\\

Explanation for Incorrect Answer B :

Choice (B) is incorrect. %uncontrolled. If one were to insert these terms into the text, the sentence would read \town lived extravagant lives; no one indulged in wild or excessive behavior.\terms should have contrasting meanings, but \that \ Explanation for Incorrect Answer C :

Choice (C) is incorrect. \were to insert these terms into the text, the sentence would read \no one indulged in wild or scrupulous behavior.\of \the second missing term should complement the meaning of \within the context of the sentence. Explanation for Incorrect Answer E :

Choice (E) is incorrect. \If one were to insert these terms into the text, the sentence would read \irreverent lives; no one indulged in wild or animated behavior.\people from being \4

ANSWERS AND EXPLANATIONS Explanation for Correct Answer E :

Choice (E) is correct. \insert these terms into the text, the sentence would read \

declining in Guatemala and Mexico was tempered by new evidence that nearby populations along the Belize River were thriving.\and the second missing term describes the current state of monkey populations. It is logical to conclude that evidence of a \disappearing.

Explanation for Incorrect Answer A :

Choice (A) is incorrect. \were to insert these terms into the text, the sentence would read \populations were declining in Guatemala and Mexico was corroborated by new evidence that nearby populations along the Belize River were prospering.\population would contradict, not \Explanation for Incorrect Answer B :

Choice (B) is incorrect. \proven to be true, and \existence. If one were to insert these terms into the text, the sentence would read \monkey populations were declining in Guatemala and Mexico was confirmed by new evidence that nearby populations along the Belize River were extant.\pulation of monkeys does not \on the study's claim.

Explanation for Incorrect Answer C :

Choice (C) is incorrect. \ing in number. If one were to insert these terms into the text, the sentence would read \were declining in Guatemala and Mexico was belied by new evidence that nearby populations along the Belize

River were dwindling.\support, not \Explanation for Incorrect Answer D :

Choice (D) is incorrect. \were to insert these terms into the text, the sentence would read \

populations were declining in Guatemala and Mexico was diminished by new evidence that nearby populations along the Belize River were debilitated.\not \5

ANSWERS AND EXPLANATIONS Explanation for Correct Answer E :

Choice (E) is correct. \sighted. If one were to insert this term into the text, the sentence would read \

disregarding the long-term welfare of the corporation.\term. It is indeed \sighted, to focus on a corporation's immediate profits at the expense of its long-term welfare.

Explanation for Incorrect Answer A :

Choice (A) is incorrect. \the text, the sentence would read \-term profits while

disregarding the long-term welfare of the corporation.\the expense of another is not necessarily \ Explanation for Incorrect Answer B :

Choice (B) is incorrect. \the sentence would read \while disregarding the long-term welfare of the corporation.\must describe the missing term, but \welfare\

Explanation for Incorrect Answer C :

Choice (C) is incorrect. \sentence would read \-term profits while disregarding the long-term welfare of the corporation.\instead of its long-term welfare is not \Explanation for Incorrect Answer D :

Choice (D) is incorrect. \the text, the sentence would read \short-term profits while disregarding the long-term welfare of the corporation.\managers to pay attention to small details while still keeping long-term goals in mind. 6

ANSWERS AND EXPLANATIONS Explanation for Correct Answer C :

Choice (C) is correct. The paragraph suggests that readers of Flaubert in Egypt should be able to \or imagine, the French author in the scenes depicted in old photographs of Egypt. Explanation for Incorrect Answer A :

Choice (A) is incorrect. It is illogical to claim that readers of Flaubert in Egypt should be able to \bring together, Flaubert.

Explanation for Incorrect Answer B :

Choice (B) is incorrect. The passage suggests that readers should be able to imagine, not \travels in Egypt while looking at the museum's collection of photographs. Explanation for Incorrect Answer D :

Choice (D) is incorrect. It is illogical to suggest that readers should be able to \ Explanation for Incorrect Answer E :

Choice (E) is incorrect. It makes little sense to say that readers should be able to \French author just outside the picture frame. 7

ANSWERS AND EXPLANATIONS Explanation for Correct Answer A :

Choice (A) is correct. The description of Flaubert \boredom was merely an act. Someone who seems to cultivate the appearance of having a certain attitude is said to be \

Explanation for Incorrect Answer B :

Choice (B) is incorrect. The author of the passage imagines Flaubert enjoying his time in Egypt, but there is no indication in the last sentence that the country \ Explanation for Incorrect Answer C :

Choice (C) is incorrect. An \the cool shade.\

Explanation for Incorrect Answer D :

Choice (D) is incorrect. The museum's photographs depict Egypt during the time period when Flaubert

presumably traveled there. No line in the passage, however, describes the French author actually posing for photographs.

Explanation for Incorrect Answer E :

Choice (E) is incorrect. While the description of Flaubert suggests that he enjoyed his time in Egypt, the last sentence does not indicate that he had any particular appreciation for the country's culture. 8

ANSWERS AND EXPLANATIONS Explanation for Correct Answer A :

Choice (A) is correct. The passage describes maps as \bare minimum\—distance, geographical features, particular patterns—effectively \maps accentuate the information that does get selected. Explanation for Incorrect Answer B :

Choice (B) is incorrect. The passage does say that maps achieve \not mean that details are made small. Rather, it means that maps avoid visual clutter by focusing on selected information. In this way, patterns can emerge more easily. Explanation for Incorrect Answer C :

Choice (C) is incorrect. According to the passage, the effect of \minimum is that patterns are created that \(lines 5-6). In other words, maps make the information they contain readily accessible and do not \momentary confusion.\

Explanation for Incorrect Answer D :

Choice (D) is incorrect. \maps sharply reduce the variety of distinctions that can be captured on them. At the same time, whatever distinctions a map does capture, it is able to capture more clearly because there is less distracting information in the map. Thus there is no reason that the distinction between words and numbers should end up being minimized if in fact both are part of the vocabulary that a map adopts. Explanation for Incorrect Answer E :

Choice (E) is incorrect. The passage mentions numbers being turned into visual images (lines 3-4), but it does not say anything about words being turned into visual images or being otherwise eliminated. The passage talks about only \eliminating things that would be useful to have. 9

ANSWERS AND EXPLANATIONS Explanation for Correct Answer C :

Choice (C) is correct. The phrase \maps the relentless torrent of information provided by the sense organs\perception have been invoked as the means by which an abundance of raw data—all that the sense organs encounter—is filtered to extract sensory information that is meaningful to humans. In other words, maps of perception allow humans to \Explanation for Incorrect Answer A :

Choice (A) is incorrect. The \perception or drawings of anything else. The maps of perception are an organizational feature of the brain that allows it to process the flood of raw data supplied by the sense organs into nonrandom patterns. The maps of perception have to do with the way data is processed, not with the organs that collect such data. Explanation for Incorrect Answer B :

Choice (B) is incorrect. Describing the phrase \actually appears to the human eye%used to understand the way that \the sense organs\of the world's appearance to the human eye do not include this information. Moreover, in the passage, maps

of perception are not limited to vision but are posited as filters of information provided by all of the sense organs.

Explanation for Incorrect Answer D :

Choice (D) is incorrect. \organs, not how the sense organs themselves (including the eye) function at the cellular level. Explanation for Incorrect Answer E :

Choice (E) is incorrect. The phrase \brain which is used in many different ways. This is different from a representation or depiction of \one person's perspective.\ people construct out of the sensory data received by the sense organs. Rather, they are tools people use in constructing those representations.

10

ANSWERS AND EXPLANATIONS Explanation for Correct Answer E :

Choice (E) is correct. The authors begin the passage by remarking that \

centuries and even over the past few decades, have not wanted it [the story of Black women] told\The authors then list some of the reasons that have been given for resisting the idea of a history that focuses specifically on Black women: \

actively suppressed it, afraid of what it would reveal. Still others have tried to deny that there was any story particular to Black women\Explanation for Incorrect Answer A :

Choice (A) is incorrect. While the authors of the passage might very well \and gender,\clear\on the exclusion of this voice from conventional accounts of women's and Black history. In lines 4-10, the authors point to the different ways this particular account (that of Black women) has been suppressed.

Explanation for Incorrect Answer B :

Choice (B) is incorrect. The purpose of lines 4-10 is to show the various objections that have been raised to treating the history of Black women as an important historical topic in its own right. Each of the objections on the list—it is not an interesting topic; it is a dangerous topic; it is not a separate topic in its own right—is presented as having been raised by a different group. Thus, although it is implied that the authors plan to discredit these views, the purpose of lines 4-10 is to present these views, not to discredit them. Explanation for Incorrect Answer C :

Choice (C) is incorrect. The historical topic that interests the authors of the passage is the topic of Black women. One purpose of the list of views presented in lines 4-10 is to show that many historians are not interested in that topic.

Explanation for Incorrect Answer D :

Choice (D) is incorrect. The views presented in lines 4-10 are the views of various groups of historians. What these different views all state is that the history of Black women is not a topic that should be pursued. There is no suggestion that any of these historians are themselves Black women. 11

ANSWERS AND EXPLANATIONS Explanation for Correct Answer A :

Choice (A) is correct. \the authors' position: \ Explanation for Incorrect Answer B :

Choice (B) is incorrect. An apology involves an admission of guilt or error of some kind. The sentence in lines 11-12 suggests that the authors of the passage feel that Black women's experiences have not been

represented in either women's or Black history. Further, they disagree with the statement that, \told the story of African Americans and the story of women, you're finished\believe that there has been an error, but it has been made by other historians, not the authors of this passage. Thus the sentence cannot be described as an apology. Explanation for Incorrect Answer C :

Choice (C) is incorrect. A decision resolves uncertainty, but there is no uncertainty present in lines 11-12. The authors do not seem to doubt that \should be told separately from the histories of African Americans and of women. Their statement that \women are different\ Explanation for Incorrect Answer D :

Choice (D) is incorrect. A concession is an acknowledgment that someone else's point is correct. The sentence in lines 11-12 does contain an acknowledgment that others hold a different point of view (\anyone may say to the contrary\flatly contradicts the view presented at the end of the previous paragraph. Explanation for Incorrect Answer E :

Choice (E) is incorrect. Although the authors of the passage do seem to criticize some historians' views at other points in the text, this is not the primary purpose of lines 11-12. The primary purpose of lines 11-12 is to forcefully state the authors' position (that \different\

12

ANSWERS AND EXPLANATIONS Explanation for Correct Answer D :

Choice (D) is correct. Although the authors acknowledge that much of Black women's experience \racism and much from sexism\This is because Black women's experiences involve \

Americans, on the one hand, and as women, on the other] in a way that chemists would call a combination, not just a mixture\transformed when they are present together\way that race and gender interact to produce Black women's unique perspective. Explanation for Incorrect Answer A :

Choice (A) is incorrect. The authors describe Black women's perspective as the result of a combination of their identities as women and as African Americans, \passage suggests, is the result of challenges that are unique to Black women. But the chemistry metaphor does not help characterize those challenges. Explanation for Incorrect Answer B :

Choice (B) is incorrect. Since the passage does not mention scientific studies of race, class, or gender, it does not make sense to say that the passage uses the chemistry metaphor to illustrate connections between such studies.

Explanation for Incorrect Answer C :

Choice (C) is incorrect. The passage does not mention any \It is concerned with making the point that the experiences of Black women—what they \still experience today\—result from a unique interaction of race and gender. The chemistry

metaphor serves to emphasize the idea that \women\Explanation for Incorrect Answer E :

Choice (E) is incorrect. The passage does not suggest that racism and sexism are similar. In particular, the chemistry metaphor does nothing to show any similarities between them. 13

ANSWERS AND EXPLANATIONS Explanation for Correct Answer B :

Choice (B) is correct. In lines 24-26 the authors say that many Black men and many White feminists \put their arms metaphorically around the Black woman's shoulder and say, 'She's with us.'\that, although Black women are sought after by both these groups, \long as they do not mention it too often\in claiming Black women as allies, but only as long as Black women can be kept from insisting on having the uniqueness of their experiences recognized. According to the authors, the figurative gesture toward Black women is both self-interested and calculating. Explanation for Incorrect Answer A :

Choice (A) is incorrect. The authors say that both Black men and White feminists \metaphorically around Black women's shoulders\way of claiming Black women for their own cause, not a gesture of affection for Black women themselves. Explanation for Incorrect Answer C :

Choice (C) is incorrect. Genuine empathy involves being aware of and sensitive to the feelings of others. But according to the authors, Black women \mention it too often\keep their mouths shut. In claiming Black women for their respective groups by putting \

metaphorically around the Black woman's shoulder\being genuinely empathetic.

Explanation for Incorrect Answer D :

Choice (D) is incorrect. There is no indication in the passage that either Black men or White feminists require any special courage to claim Black women for their respective groups. The suggestion is rather that both groups claim Black women out of self-interest. Explanation for Incorrect Answer E :

Choice (E) is incorrect. \the reason many Black men and many White feminists \Black woman's shoulder and say, 'She's with us'\women's interests, there is no suggestion that this gesture is arrogant. The authors of the passage do suggest

that the people who make this gesture are being insensitive to the different experiences of Black women, but they do not claim that the gesture itself is overtly or openly arrogant. 14

ANSWERS AND EXPLANATIONS Explanation for Correct Answer D :

Choice (D) is correct. The \nervous and guilty\makes it so difficult for us as a nation to get rid of our fear that difference implies, even guarantees, animosity and opposition\e's fear that focusing on differences will lead to disharmony. Explanation for Incorrect Answer A :

Choice (A) is incorrect. The \feel nervous and guilty\fear that focusing on differences will lead to disharmony. Further, two of the groups discussed in the passage, Black political leaders and White feminist political leaders, are specifically working for political change. Thus it does not make sense to describe resistance to political change as the explanation for the \mentioned in line 32.

Explanation for Incorrect Answer B :

Choice (B) is incorrect. The passage does not mention role models or people's fears of being held up as role models.

Explanation for Incorrect Answer C :

Choice (C) is incorrect. On the contrary, the authors suggest that, at least with respect to their attitudes toward Black women, Black political leaders and White feminist political leaders behave very similarly. There are no references to other people who might be considered to hold positions of power. Thus it is unlikely that the authors regard differences in the ways men and women behave in positions of power as the explanation for the “sad situation.\

Explanation for Incorrect Answer E :

Choice (E) is incorrect. The \feel nervous and guilty\fear that focusing on differences will lead to disharmony. This explanation depends on conformity in a specific respect: people being reluctant to acknowledge difference. Thus placing a high value on nonconformity—that is, on being different from other people—cannot explain the \ 15

ANSWERS AND EXPLANATIONS Explanation for Correct Answer E :

Choice (E) is correct. \people in the United States are uncomfortable about acknowledging differences—e.g., between Black women and White women—because of \(lines 36-37). In lines 37-39 the authors then comment on this refusal to accept difference: \difference is not the road to harmony. It is the road only to a kind of false unity that is so fragile it will splinter at a touch.\disapproved of and warned against.

Explanation for Incorrect Answer A :

Choice (A) is incorrect. To \or to tell in secret. In lines 37-39, the authors do not tell anything in confidence. Rather, they are making a bold statement about the impact of the sort of \(line 38-39) they describe. Further, the authors make it clear that they disapprove of the practice of denying difference in an effort to maintain unity. They do not seem concerned with potential criticism of their views, nor do they seem to desire to keep their views secret. Thus the tone of lines 37-39 cannot be described as confiding.

Explanation for Incorrect Answer B :

Choice (B) is incorrect. \their position that the \-37) is a mistaken fear. The tone is measured and even emphatic (\confrontational or challenging, so the tone of these lines cannot be described as defiant. Explanation for Incorrect Answer C :

Choice (C) is incorrect. \their position that the \is a mistaken fear. But there is no suggestion that they are unwilling to accept that such fear exists, or that they are doubtful that people seek to deny difference because of that fear. Thus the tone of lines 37-39 cannot be described as skeptical.

Explanation for Incorrect Answer D :

Choice (D) is incorrect. \surrender. In lines 37-39, there is no indication that the authors have given up on their attempt to show that denying difference is a bad strategy for securing harmony among people.

16

ANSWERS AND EXPLANATIONS Explanation for Correct Answer A :

Choice (A) is correct. Elsa Barkley Brown is quoted as saying that history is \rhythms being played simultaneously\and inclusion (\Explanation for Incorrect Answer B :

Choice (B) is incorrect. Barkley Brown is quoted as saying that the \occur in isolation but in dialogue with a myriad of other people and events\might possibly suggest mutual respect, but it does not emphasize privacy. Explanation for Incorrect Answer C :

Choice (C) is incorrect. Barkley Brown's view that \isolation but in dialogue with a myriad of other people and events\interdependence of people, not their self-reliance or individuality. Explanation for Incorrect Answer D :

Choice (D) is incorrect. Barkley Brown's image of a \and all the stories interrelate and play off each other\more than it suggests a simple acceptance, at face value, of what other people have to say. Explanation for Incorrect Answer E :

Choice (E) is incorrect. Barkley Brown's image of \simultaneously\invention.

17

ANSWERS AND EXPLANATIONS Explanation for Correct Answer B :

Choice (B) is correct. The narrator originally begins the story by saying \stops short of identifying the department. By way of an explanation, the narrator mentions the \of a department. Then the narrator speaks of avoiding \this time beginning with \\the department might have been. Explanation for Incorrect Answer A :

Choice (A) is incorrect. The narrator does not display any anger in the story, and thus does not use strong language inspired by anger. The reference to \e 3 concerns the angry reaction of government offices at finding uncomplimentary stories told about them; it does not describe the narrator. Explanation for Incorrect Answer C :

Choice (C) is incorrect. The narrator does not mention any arguments between different government offices. The wrath of government offices referred to in line 3 is not anger directed by one government office at another, but anger at anyone who would write negative things about a government office while at the same time fully identifying it.

Explanation for Incorrect Answer D :

Choice (D) is incorrect. In lines 33-42 the narrator describes the teasing that Akaky was subjected to by the young clerks in his department. But the word \teasing of Akaky. Rather, it refers to what might happen to a writer if the writer were to identify a government office when writing unflattering things about it. Explanation for Incorrect Answer E :

Choice (E) is incorrect. The narrator is about to identify (or pretends to be about to identify) the government department in which the story is set but then decides not to. The reason for not naming the department is only hinted at: the department would be moved to wrath, and there would be unpleasantness. However, the strong implication is that the person who would suffer unpleasant consequences is just the narrator. There is no indication that the unpleasantness referred to in line 5 would be discomfort felt by many Russian citizens. 18

ANSWERS AND EXPLANATIONS Explanation for Correct Answer C :

Choice (C) is correct. Akaky is described as being \19-20). In fact, no one remembers \superiors jokingly sum up this impression by saying that \councillor\

Explanation for Incorrect Answer A :

Choice (A) is incorrect. There is no evidence in the story that Akaky ever tried to escape his basic nature. The narrator suggests that there was only one time when Akaky did something that was slightly more challenging than his usual daily routine, but that was at instructions from a director of the department, not on his own initiative.

Explanation for Incorrect Answer B :

Choice (B) is incorrect. The department officials say, jokingly, that \councillor in uniform all complete and with a bald patch on his head%uniform is part of their joke, but there is no reason to think that they would have thought of Akaky any differently if he had not worn that particular uniform. Explanation for Incorrect Answer D :

Choice (D) is incorrect. The narrator makes clear that no one in the department could remember a time when Akaky had not been working there. Explanation for Incorrect Answer E :

Choice (E) is incorrect. The remark that \complete and with a bald patch on his head\Akaky's real age, nor is the issue of Akaky's age touched on anywhere else in the story. 19

ANSWERS AND EXPLANATIONS Explanation for Correct Answer D :

Choice (D) is correct. The \the building where Akaky worked would not have taken any notice of. The point of comparing Akaky to a simple fly is to emphasize how completely the porters ignored Akaky when he came in. So the fly serves as an image of something that is easily overlooked. Explanation for Incorrect Answer A :

Choice (A) is incorrect. A fly may be annoying at times, but the comparison in lines 24-25 is not based on this quality of flies.

Explanation for Incorrect Answer B :

Choice (B) is incorrect. The point of calling the fly \rather that there is nothing about a fly that would attract special attention. Explanation for Incorrect Answer C :

Choice (C) is incorrect. A \slow-moving one, but this would not be the quality that the comparison in line 25 appeals to. Explanation for Incorrect Answer E :

Choice (E) is incorrect. The mention of the fly serves to stress just how little notice the porters took of Akaky. So the point of referring to the fly cannot be to introduce an image of something potentially harmful. 20

ANSWERS AND EXPLANATIONS Explanation for Correct Answer A :

Choice (A) is correct. The two quotations in lines 28-29 are examples of expressions that are, according to the narrator, commonplace \behaving politely in the ordinary course of carrying on routine business. Explanation for Incorrect Answer B :

Choice (B) is incorrect. The two quotations in lines 28-29 are not examples of compliments, but rather of everyday civilities.

Explanation for Incorrect Answer C :

Choice (C) is incorrect. The narrator clearly does not view these expressions as examples of unreasonable demands, but rather of everyday civilities. Explanation for Incorrect Answer D :

Choice (D) is incorrect. The two quotations in lines 28-29 actually do serve as examples of small talk, but not of unappreciated small talk. The narrator says that in Akaky's case, the clerk's assistant who gave him his work did not say any of these things to Akaky. The narrator then comments, \suggesting that Akaky would actually have appreciated having those things said to him. Explanation for Incorrect Answer E :

Choice (E) is incorrect. These expressions may be \not really explain anything. They serve as examples of the kind of courtesies that are exchanged by people who have at least some respect for one another but which Akaky's coworkers did not extend to him. 21

ANSWERS AND EXPLANATIONS Explanation for Correct Answer C :

Choice (C) is correct. To say that people told \stories that they had made up, invented, or \ Explanation for Incorrect Answer A :

Choice (A) is incorrect. If \invention\the narrator says that the stories the young clerks told about Akaky were the product of any kind of experimentation.

Explanation for Incorrect Answer B :

Choice (B) is incorrect. It is true that, in technological contexts, what is called an \device, but the context in which \ion\context of making up stories.

Explanation for Incorrect Answer D :

Choice (D) is incorrect. While \the clerks fabricate rather than \ Explanation for Incorrect Answer E :

Choice (E) is incorrect. The word \a story: the greater the skill, the better the story. But the quality of the stories that the young clerks fabricate and tell is not relevant.

22

ANSWERS AND EXPLANATIONS Explanation for Correct Answer A :

Choice (A) is correct. There are indications that the narrator disapproves of the young clerk's cruelty. In lines 11-15, the narrator points out that Akaky is in a category of workers who are among \themselves.\the reaction of the one young clerk who has not yet learned to tease Akaky. Without any strong direct expression of disapproval, the narrator nevertheless manages to convey disapproval. Explanation for Incorrect Answer B :

Choice (B) is incorrect. There is no indication in the narrative that Akaky is the supervisor of the young clerks, and there is no indication that they are disrespectful of any co-workers aside from Akaky. Explanation for Incorrect Answer C :

Choice (C) is incorrect. The narration does suggest that the young clerks do not all work hard all of the time. But even though their actions are described in some detail, the narrator never says anything to the effect that they should be doing their assigned work instead of tormenting Akaky. Explanation for Incorrect Answer D :

Choice (D) is incorrect. The young clerks do seem to lack a challenge, but the author does not pretend to have any sympathy with them for any such lack. His sympathy—real sympathy—is squarely on Akaky's side, even though the expression of this sympathy is not very direct. Explanation for Incorrect Answer E :

Choice (E) is incorrect. There is also indication that the narrator is in the least amused. 23

ANSWERS AND EXPLANATIONS Explanation for Correct Answer E :

Choice (E) is correct. The clerk who is new to the office is described as being \Akaky finally responds to his tormentors with words. This suggests that he is responding to Akaky's words with compassion.

Explanation for Incorrect Answer A :

Choice (A) is incorrect. The passage does not indicate that the clerk who is new to the office is confused about anything.

Explanation for Incorrect Answer B :

Choice (B) is incorrect. There is no indication that the clerk who is new to the office responds to Akaky's words with sarcasm or bitter, cutting language that is used to hurt someone's feelings. The clerk does not say anything aloud, and there is no evidence that he feels bitterness. Explanation for Incorrect Answer C :

Choice (C) is incorrect. The new clerk cannot have had any difficulty believing that Akaky said what he did. Given the taunting behavior of the other young clerks, for Akaky to say, \me?\

本文来源:https://www.bwwdw.com/article/xbxr.html

Top